Classifying Extrema and Saddle Points of Multivariable Functions

Click For Summary
The discussion focuses on finding and classifying relative extrema and saddle points for the function f(x, y) = xy - x^3 - y^2. The critical point identified is (0, 0), where the determinant D is calculated as -1, indicating D < 0. This leads to the conclusion that (0, 0) is a saddle point, regardless of the value of fxx at that point. Another critical point mentioned is (1/6, 1/12), but the main focus remains on the classification of (0, 0). The consensus is that D < 0 confirms the saddle point classification.
Doug_West
Messages
9
Reaction score
0

Homework Statement



Find and classify all relative extrema and saddle points of the function
f(x; y) = xy - x^3 - y^2.

Homework Equations



D = fxx *fyy -fxy^2

The Attempt at a Solution



I got D < 0 where D = -1 and fxx = 0, when x=0 and y=0. However I am unsure as to the conclusion I should arrive at when D < 0 but fxx = 0. I'm thinking that this is a saddle point?

Thanks for the help in advance,
Dough
 
Physics news on Phys.org
Doug_West said:

Homework Statement



Find and classify all relative extrema and saddle points of the function
f(x; y) = xy - x^3 - y^2.

Homework Equations



D = fxx *fyy -fxy^2

The Attempt at a Solution



I got D < 0 where D = -1 and fxx = 0, when x=0 and y=0. However I am unsure as to the conclusion I should arrive at when D < 0 but fxx = 0. I'm thinking that this is a saddle point?

Thanks for the help in advance,
Dough
You need to find locations where both of the 1st partial derivatives are zero.
 
yep those two pts are

x=0, y=0
and
x=1/6 y=1/12
 
evaluating pt 0,0 I get D<0 where D= -1 and fxx(0,0) = 0, so then this is a saddle point. However is it a saddle point because D<0 or because fxx = 0?
 
guess ur doing mab127 too

when d < 0 its a saddle point doesn't matter what fxx is
 
haha yep :D, thanks for the help.
 
Question: A clock's minute hand has length 4 and its hour hand has length 3. What is the distance between the tips at the moment when it is increasing most rapidly?(Putnam Exam Question) Answer: Making assumption that both the hands moves at constant angular velocities, the answer is ## \sqrt{7} .## But don't you think this assumption is somewhat doubtful and wrong?

Similar threads

  • · Replies 12 ·
Replies
12
Views
2K
  • · Replies 8 ·
Replies
8
Views
4K
  • · Replies 3 ·
Replies
3
Views
3K
Replies
3
Views
14K
Replies
5
Views
6K
  • · Replies 2 ·
Replies
2
Views
2K
Replies
4
Views
2K
Replies
5
Views
3K
  • · Replies 3 ·
Replies
3
Views
2K
Replies
22
Views
3K